Using Abel's Theorem, find the Wronskian

Click For Summary
The discussion focuses on applying Abel's Theorem to find the Wronskian of two solutions for the second-order linear ODE x'' + 1/sqrt(t^3)x' + t^2x = 0 for t > 0. Participants confirm that the integral of 1/sqrt(t^3) leads to a Wronskian expression of W(t) = C e^{- \int t^{-3/2} dt}, with the integral evaluated as -2t^{-1/2}. The conversation highlights that while natural logarithms often appear in similar problems, they are not a strict requirement in this case. Emphasis is placed on careful handling of negative signs and the use of TeX for clarity. Overall, the solution presented is deemed valid and correctly follows Abel's formula.
NiallBucks
Messages
7
Reaction score
0
Member warned about posting without the homework template
Using Abel's thrm, find the wronskian between 2 soltions of the second order, linear ODE:
x''+1/sqrt(t^3)x'+t^2x=0
t>0


I think I got the interal of 1/sqrt(t^3) to be 2t/sqrt(t^3) but this is very different to the other examples I've done where a ln is formed to cancel out the e in the formula
W(x1,x2)= Cexp(-intergral 1/sqrt(t^3) dt)
 
Last edited:
Physics news on Phys.org
##p(t) = \frac{1}{\sqrt{t^3} }, q(t) = t^2## fits Abel's formula.
So:
##W(t) = C e^{- \int t^{-3/2}dt } ##
In your solution to the integral, I assume you meant that: ## - \int t^{-3/2}\, dt = 2t^{-1/2} ##. This looks to be the best way to answer the problem.
You will often see the natural log occur in practice problems, but there is no rule stating that it will always be there.
A common format looks like ##p(t) = \frac{f'(t)}{f(t)},## so the integral ends up as ##\ln f(t)##, so ##e^{- \ln f(t) } = \frac{1}{f(t)}.## But that is not the format you have here. It looks like your solution is good. Remember to be careful with your negative signs, and try to use Tex when possible to make reading easier.
 
  • Like
Likes NiallBucks
Question: A clock's minute hand has length 4 and its hour hand has length 3. What is the distance between the tips at the moment when it is increasing most rapidly?(Putnam Exam Question) Answer: Making assumption that both the hands moves at constant angular velocities, the answer is ## \sqrt{7} .## But don't you think this assumption is somewhat doubtful and wrong?

Similar threads

Replies
7
Views
2K
Replies
2
Views
1K
Replies
1
Views
2K
  • · Replies 1 ·
Replies
1
Views
2K
Replies
3
Views
3K
  • · Replies 4 ·
Replies
4
Views
2K
  • · Replies 105 ·
4
Replies
105
Views
6K
  • · Replies 1 ·
Replies
1
Views
2K
Replies
2
Views
1K
  • · Replies 7 ·
Replies
7
Views
2K